КОМЕНТАРІ •

  • @bprpcalculusbasics
    @bprpcalculusbasics 5 місяців тому +22

    Get your indeterminate cat t-shirt: 👉 amzn.to/3qBeuw6

  • @gonzalotapia1250
    @gonzalotapia1250 5 місяців тому +863

    DO NOT BE CONFUSED!
    The number 1, to the power of infinity, is 1.
    A number that approaches 1, to the infinity, can be anything.

    • @PhilosophicalNonsense-wy9gy
      @PhilosophicalNonsense-wy9gy 5 місяців тому +102

      1^∞ = 1?????????

    • @Sg190th
      @Sg190th 5 місяців тому +103

      @@PhilosophicalNonsense-wy9gy It's always 1 if you mutliply 1 by itself as many times as you want.

    • @arimermelstein9167
      @arimermelstein9167 5 місяців тому +156

      @@PhilosophicalNonsense-wy9gy if it’s the constant 1, then yes. But if it’s some quantity that approaches 1, then it may or may not be. We need to do more work to see.

    • @OkayFine-ie5pm
      @OkayFine-ie5pm 5 місяців тому +20

      ​@@PhilosophicalNonsense-wy9gyyup that's totally correct
      And I can bet my life on this
      The real comment in 100%
      As from left hand side.. you can see (1-)^(♾️)->0
      (1+)^{♾️}->♾️
      And it's at a state all upto limits

    • @bprpcalculusbasics
      @bprpcalculusbasics 5 місяців тому +78

      @@PhilosophicalNonsense-wy9gy yes, please see 3:40

  • @bdulAhad0
    @bdulAhad0 3 місяці тому +14

    That is some quick marker switching at 6:03!!

  • @ianfowler9340
    @ianfowler9340 5 місяців тому +82

    One of the intuitive ways of understanding things like (1+1/x)^x is to think about the "rate" that base --->1 and exponent ---> inf. They are, in a sense, competing against each other. In the end you will be using L'Hopital's rule after switching the ln and the limit.. The ln / lim switch turns an exponentiation to a multiplication: x * ln(1+1/x) Turn that into a ratio moving the 1/x into the denom.. So now you can compare the "rates" (since you now have a ratio) at which both num. and denom. approach 0 by applying L'Hopital to ln(1+1/x) and 1/x.
    You compare the rates by looking at the ratio f '/g ' - which is ultimately what L'Hopital does. If the ratio of their rates ( f '/g ' ) converges so then will f/g. The inverse of the ln will get you the limit of the original exponentiation. In this case, after L'Hopital, you pick up a factor of (-1/x^2) in both num. and denom. which helps you to simply and then compute the limit of (f ' / g ' ).
    If (f ' /g ') blows up, then so will f/g.

    • @notmuchgd9842
      @notmuchgd9842 5 місяців тому

      that second paragraph suddenly made l'hopital make sense to me, i can see the justification behind it now

    • @ZqTi0
      @ZqTi0 4 місяці тому

      I think an easy to figure out why the answer is not 1 is if we simply add it together. 1+1/3x = (3x+1)/3x. If we compare the numerator and denominator we can see that as x gets bigger and bigger the numerator is always a little bit bigger than the denominator, which means that the whole expression will not be exactly one as it approaches positive Infinity.

  • @jamirimaj6880
    @jamirimaj6880 2 місяці тому +5

    Another way to think about it: 1.000000000000000001 raised to x goes to infinity as x goes to infinity as well.

  • @cyrusyeung8096
    @cyrusyeung8096 5 місяців тому +549

    However, why does the limit converge in the first place? My high school teacher told me that (1 + 1/n)^n -> e as n -> +∞ *by definition* , but she never told me why it converges.

    • @rohanagrawal6209
      @rohanagrawal6209 5 місяців тому +143

      e is the base exponent such that derivative equals itself. If you put this constraint in the limit definition of the derivative, you will obtain the limit definition for e.

    • @danielemacheda4493
      @danielemacheda4493 5 місяців тому +111

      You can indeed prove that the limit does converge to e however the proof IIRC is long and it is generally not teached in high schools.
      Typically they will assume that the limit converges to e as a fact and use that to explain the convergence of other limits such as ln(1+x)/x=1 as x goes to 0

    • @olegsea
      @olegsea 5 місяців тому +104

      1) it monotonically increases
      2) has an upper bound
      therefore, it converges.
      (1) can be proven if you use Newton's binomial theorem for n and n+1 and compare them
      (2) can be proven using the above thing + the fact that 1/k!

    • @pneujai
      @pneujai 5 місяців тому +30

      consider the sequence Eₙ=(1+1/n)ⁿ, and you can prove that it is monotonic increasing, that is Eₙ₊₁≥Eₙ (hint: AM≥GM).
      Then prove it bounded above by 3.
      Using MCT, Eₙ converges

    • @pneujai
      @pneujai 5 місяців тому +9

      you can also observe (not prove) this by substituting large numbers into n with your calculator to see that it converges to e

  • @shreyjain3197
    @shreyjain3197 Місяць тому +7

    Use the formula: lim (1+f(x))^g(x) as f(x) approaches 0 and g(x) approaches infinity = e^lim f(x)g(x) as f(x) approaches 0 and g(x) approaches infinity

  • @KazACWizard
    @KazACWizard 5 місяців тому +8

    i used the exponential property rewriting the function as e to the ln of the function. then brought the x to denomintor as 1/x and applied l hopitals rule once to obtain the cube root of e

  • @jeffreyblack666
    @jeffreyblack666 5 місяців тому +9

    I would take a simpler approach.
    If you look at a binomial expansion of (1+x)^n, then the first term is 1, and the second term is n*x.
    Using that here, the first term is 1, the second term is x/3x = 1/3. That means that overall, the result will be greater than 1+1/3, so it can't be equal to 1.

  • @Aristothink
    @Aristothink 5 місяців тому

    Great video as always. I love to watch your videos. Here, on this limit I would think like this: the "x" inside the parenthesis is bringing the number inside the parenthesis down, BUT the x outside the parenthesis is bringing the final number up. Since one is bringing the number down and the other is bringing the number up, they are fighting against each other, so nothing can be said just by replacing "x" by zero. That is why it's an indeterminate limit.

  • @lorenzogiampietri6812
    @lorenzogiampietri6812 5 місяців тому +5

    That’s surprisingly interesting, never thought about it

  • @user-bl5lk7jn7f
    @user-bl5lk7jn7f 5 місяців тому +21

    also you can do this, whenever the there is a exponent and you have a limit like lim f(x)^g(x) = e ^(lim (f(x) -1) g(x))
    this would give us e^lim x->infinity(1+1/3x -1)(x) = e^1/3

    • @mufaddalrajkotwala
      @mufaddalrajkotwala 2 місяці тому +1

      Absolutely this is the easiest way to solve this question in literally 2 steps

    • @voidete7793
      @voidete7793 Місяць тому

      do you have the demo for this formula please?

    • @user-bl5lk7jn7f
      @user-bl5lk7jn7f Місяць тому

      @@voidete7793 i have already written the formula in thr comment
      For the demo, you can try using it in the same question as in the video

  • @evanevman8852
    @evanevman8852 Місяць тому

    When I begin calculus, I am certain I will be watching a lot of your videos. Good content!

  • @guythat779
    @guythat779 4 місяці тому

    Great video
    Had to rejog my brain on this problem, it's deceptively hard but as soon as you wrote the e limit (which i already knew and thought of but seeing it is different) everything clicked

  • @rupom_1670
    @rupom_1670 3 місяці тому +1

    oh my god
    I started to do calculus and i was stuck in a problem like this
    (it was in the eulers numbers limit lol)
    this video cleared my confusion and i just want to say thanks for the clear explanation

  • @arimermelstein9167
    @arimermelstein9167 5 місяців тому +2

    Another, more general way to do this is to take logs, rewrite it as 0/0 or inf/inf, solve that limit and then take it back to the original. That’s how I did this one.
    Your way is more elegant and I learned something new!

  • @JL-pc2eh
    @JL-pc2eh 5 місяців тому +1

    love the reddit question videos

  • @Sg190th
    @Sg190th 5 місяців тому +20

    Beauty of algebraic manipulation. I had a feeling it was related to e.

  • @MasterCatto1912
    @MasterCatto1912 2 місяці тому

    I recently studied a formula for this exact situation where 1 is raised to infinity if you try to put in the limit. The format being lim(f(a))^g(x)
    x->a such that f(a)=1 and g(a)->∞. Then the above limit can be rearranged into the format e^{lim (g(x))[f(x)-1]}
    x->a

    • @voidete7793
      @voidete7793 Місяць тому

      Do you have the demo for it please?

  • @nexu9568
    @nexu9568 5 місяців тому +4

    UA-cam keeps recommending you to me. And tbh as someone who loves maths I finally gotta subscribe to you.

  • @sergioborghesan6318
    @sergioborghesan6318 5 місяців тому

    Thank you Prof!

  • @battle00333
    @battle00333 5 місяців тому +2

    Would this make more sense if you changed 1 + 1/3x into (3x + 1)/3x ?
    Cus then if x= 10, then we get (3*10 +1)/3*10 = (31/30)^10
    for x=1000 we would get (3*1000+1)/3*1000 = (3001/3000)^1000
    for x = 1000000 -> (3000001 / 3000000)^(10^6)
    etc.

  • @StereoSpace
    @StereoSpace 4 місяці тому +5

    When you become a professional in any field, you see things immediately that those learning might take hours to see, or not see at all.

  • @anonakkor9503
    @anonakkor9503 5 місяців тому

    had a similar limit but never understood why…. Now I get it thx!!!

  • @Avighna
    @Avighna 5 місяців тому +5

    MATH. IS. BEAUTIFUL.

  • @AlessioVragnaz98
    @AlessioVragnaz98 Місяць тому

    Wow i never thought about this

  • @ToddKunz
    @ToddKunz 5 місяців тому +2

    That is brilliant. I did the problem before watching the video and I got 1 as an answer...of course, I was wrong. Thank you so much for doing this interesting problem.

  • @user-cr1wn8oe1b
    @user-cr1wn8oe1b 5 місяців тому +1

    Can we use the logarithmic differentiation rule?
    After that we use L'hôpital rule ?

  • @_blankato9336
    @_blankato9336 5 місяців тому

    Can you just multiply the exponent by 3/3 and get the same result? I dont remember limits very well.

  • @patsk8872
    @patsk8872 3 місяці тому +1

    Can also take ln of each side and use the rule that lim ln = ln lim

  • @panyachunnanonda6274
    @panyachunnanonda6274 Місяць тому

    I love this question.

  • @ManjulaMathew-wb3zn
    @ManjulaMathew-wb3zn 5 місяців тому

    I got the same answer by taking ln of the limit then converting it to a ratio and applying La Hospitals rule.

  • @Physicsnerd1
    @Physicsnerd1 Місяць тому

    Always interesting.

  • @RandyWong
    @RandyWong 5 місяців тому +3

    Low key tempted to get some of that product drop on EXPO markers. 😅

  • @AlfonsoNeilJimenezCasallas
    @AlfonsoNeilJimenezCasallas 5 місяців тому

    cool T-shirt, master!

  • @paualberti6667
    @paualberti6667 5 місяців тому

    I was taught that when lim a^b = 1^inf then lim a^b = e^(lim b(a-1))

  • @zachansen8293
    @zachansen8293 3 місяці тому

    When you look at these questions it's clear that as one term goes towards zero, it's contribution to the overall answer also increases. So it's obvious why you can't just treat the term on its own as zero -- because its contribution is modified later by something that increases the value at a relatively corresponding rate. If it were just 1/3x as x->0 then yeah, it's zero.

  • @speakingsarcasm9014
    @speakingsarcasm9014 5 місяців тому +2

    The base is slightly larger than 1. Consider something like (1+0.1)^10, it's not 1, there's a rate of interest 10% compounded over 10 years :)

  • @SuperTommox
    @SuperTommox 5 місяців тому

    Very interesting question

  • @paull244
    @paull244 5 місяців тому +1

    beautiful limit !

  • @xzaratulx
    @xzaratulx 4 місяці тому

    Ha, here I really thought it approaches 1.
    But the ever increasing exponent is not to be underestimated.

  • @RiboTheGreat
    @RiboTheGreat 5 місяців тому +6

    This is true but a general formula exists for 1^infinity problems
    Lim x->a ((f(x))^g(x))=e^(Lim x->a (f(x)-1)*g(x))

    • @voidete7793
      @voidete7793 Місяць тому

      do you have the demo for it please?

  • @GameChangerkun
    @GameChangerkun 5 місяців тому

    Can anyone help me with simplifying e^(cosx)=-2sin(x)

  • @user-oy4qs3ty8h
    @user-oy4qs3ty8h 5 місяців тому +3

    e^1/3

  • @EdwardCurrent
    @EdwardCurrent 5 місяців тому +2

    Confused at 3:55 -- the shirt says that 1 ^∞ is an indeterminate form, so how does it = 1?

    • @benseb2512
      @benseb2512 5 місяців тому +3

      because the first time it approaches 1 whereas the second time it is exactly one. You cannot say for certain whether a number approaching 1 to the infinity power is equal to 1

    • @xBrokenMirror2010x
      @xBrokenMirror2010x 5 місяців тому +1

      Its because 1, in this case, is actually 1 plus 1/x where x approaches infinity
      As long as the rest of the problem is finite, it is effectively equal to 1(but not exactly equal to 1, its essentially equal to 1.0000...0001 where there is an uncountable infinity of 0's between the "...". No matter how large the number you multiply (1+1/infinity) by is, you will still treat it as 1x, until the number you are multiplying by is also approaching infinity.

    • @EdwardCurrent
      @EdwardCurrent 5 місяців тому

      @@benseb2512 So the mention on the shirt is specific to the limit, and strictly speaking 1^∞ is not an indeterminate form?

    • @phiefer3
      @phiefer3 5 місяців тому +6

      @@EdwardCurrent 1^inf is indeterminate, always.
      However, we evaluate indeterminate forms all the time, within the context of a given limit of a function. When he says that "exactly 1"^inf is 1, what he means is that the limit of the function 1^x as x->inf is 1. This is the evaluation of a specific limit of a specific function, and so it doesn't necessarily apply to other limits that have the 1^inf form.
      It's the same as if you have the function x/x and take the limit as x->inf. This is an inf/inf situation, which is indeterminate. However, we can use several methods to show that the value of this limit is 1. So in this situation inf/inf =1, however, for other limits of other functions that end up in an inf/inf situation the solution may be different.
      That's essentially what indeterminate means: that there is not a single generalized solution, and that other methods must be used in order to evaluate them in the context of a given function/limit. So in general 1^inf is indeterminate. But in the specific context of 1^x as x->inf then it is equal to 1.

    • @EdwardCurrent
      @EdwardCurrent 5 місяців тому +1

      @@phiefer3 Very well explained, thank you

  • @sukritshankar03
    @sukritshankar03 Місяць тому

    [ For 1^Inf form, one should learn to convert to a 0 x Inf form by taking ln, and then rewriting in 0/0 form to apply L'Hospital's Rule ] Let the limit be y. Then taking ln (and exchanging lim and ln on RHS), we have ln(y) = lim_x-->Inf x ln (1 + 1/3x) = lim_x-->Inf (ln(1 + 1/3x))/(1/x) This is now 0/0 form, so let us apply L'Hospital's Rule. We will get ln(y) = lim_x-->Inf (1/3 (-1/x^2))/ ((1 + 1/3x)(-1/x^2)) = lim_x-->Inf (1/3) / (1 + 1/3x) and here now we can put x = Inf to get ln(y) = 1/3, which implies y = e^(1/3).

  • @sirjanabhatt
    @sirjanabhatt 5 місяців тому

    Nice short 😅

  • @guydesautels
    @guydesautels 2 місяці тому

    1 + 1/3000= 1.003 1.0003 ; however does not affect video in question.

  • @thenetsurferboy
    @thenetsurferboy 5 місяців тому

    Very good

  • @F1r1at
    @F1r1at 5 місяців тому +5

    So the thing is: you can replace 1/3x with 0.
    Just need to remember that all 0s and all infinities in the limits are not numbers. They're variables. Infinity has a really, really big number as its value, and 0 has really, really small number as its value. And they're written like that just for simplicity.
    So while you can do something like: 1 + 1/3x = 1 + 0. You can't do 1 + 0 = 1.

  • @christophertitanium8424
    @christophertitanium8424 5 місяців тому

    I'm a dumb dumb welder with a caveman brain, and no practical application for this level of math. That being said, I enjoy your videos and your teaching style. Plus that two marker technque is pretty slick!

  • @ramziabbyad8816
    @ramziabbyad8816 5 місяців тому

    Actually, a^x is not continous at infinity as neighborhoods of infinity actually grow larger on the riemann sphere, lol...

  • @lirantwina923
    @lirantwina923 5 місяців тому

    Please try to integrate 1/x^5+1

  • @pumkin1319
    @pumkin1319 5 місяців тому

    Wouldnt a = 1 and b = 1/3 for the last formula?? It comes to the same conclusion but i just want to know if im tripping

    • @telanis9
      @telanis9 5 місяців тому

      If you look at the equation after he subbed t for x, yes. But he did it for the original form of the equation. Remember t and x are just labels, they can be anything and don't actually matter -- they're only different so that we can tell them apart.

  • @williamn3070
    @williamn3070 5 місяців тому +9

    why does lim of (1+ 1/t)^t, t--> infinity = e?

    • @borstenpinsel
      @borstenpinsel 5 місяців тому +5

      Why is anything e. It's so annoying :D in every math problem there is one step "oh yeah x/y+z^w = e" which is also cos(x)/sqrt(x)*whatyouhadforbreakfast and incidentally it's also everybody's length if intestines divided by the weight of their nose 😅

    • @amritlohia8240
      @amritlohia8240 5 місяців тому

      @@borstenpinsel @williamn3070 You can show that (1+1/t)^t is monotonically increasing in t (using e.g. Bernoulli's inequality) and is bounded above (using e.g. the binomial theorem), so it converges by the monotone sequences theorem. We then define the limit to be e, and you can prove that this is equivalent to any other definition of e you might have had: e.g. to prove it's equivalent to the power series definition, you can again use the binomial theorem and some basic algebraic manipulations (see e.g. courses.maths.ox.ac.uk/pluginfile.php/93465/mod_resource/content/1/supplementary%20notes%20on%20e%20by%20Priestley.pdf).

    • @rayane--21pilots8
      @rayane--21pilots8 4 місяці тому +1

      ​@@borstenpinselbro let me have a sip of your drink

    • @ItamarGlikman
      @ItamarGlikman 4 місяці тому

      Thats literally the definition of e...

    • @amritlohia8240
      @amritlohia8240 4 місяці тому

      @@ItamarGlikman It's *one possible* definition of e. Another common definition is the sum of 1/n! for n >= 0, but it's not straightforward to show that these definitions are equivalent.

  • @amaanmuhammed
    @amaanmuhammed 5 місяців тому +2

    5:24 or is it??....🤨

  • @mateusmorais4056
    @mateusmorais4056 3 місяці тому

    Excelent!

  • @Zx-Chatgarou
    @Zx-Chatgarou 4 місяці тому

    Give me this shirt. We needs this

  • @jonathandawson3091
    @jonathandawson3091 5 місяців тому

    More importantly, keeping aside how easy it is to see it is e^(1/3), can anyone tell my why it is tagged as differential calculus?

    • @robertlunderwood
      @robertlunderwood 3 місяці тому

      Because limits are taught in a differential calculus course.

  • @anerwyn9803
    @anerwyn9803 5 місяців тому

    Whenever I see infinity expressed mathematically I always treat it as a sorta variable myself, because it can be any positive number really....infinity just means a really really really big number after all.

    • @lugia8888
      @lugia8888 2 місяці тому

      No, it is not a number. That is why we have a whole chapter on limits! “Really big number” implies it is finite.

  • @Migu6
    @Migu6 5 місяців тому +1

    Rather than explain it in terms of indeterminate forms, I would say that lim f(x)^g(x) = (lim f(x))^(lim g(x)) only works if lim f(x) and lim g(x) both converge. In this case lim g(x) doesn't converge.

  • @augusto256
    @augusto256 5 місяців тому +1

    Cool t-shirt 👕 !!!

  • @MrSantaForever
    @MrSantaForever Місяць тому

    This limit is a definition of e, and he literally writes "not e" 4:37

  • @Anuj_Chaudhary692
    @Anuj_Chaudhary692 5 місяців тому

    It's 1 power infinity type of limit so easy

  • @iulianhodorog9979
    @iulianhodorog9979 5 місяців тому

    I suspect it's 3rd root of e..

  • @RadoslavFicko
    @RadoslavFicko 5 місяців тому

    Let the heat losses of one machine be E=E(1-n) and let each successive machine capture the energy losses from the previous machine, so that the total losses are E=E(1-n).(1-n).....=E(1-n)^x. If the efficiency of the machines is n=1/x and x approaches infinity, then the efficiency of one machine will be zero, but the efficiency of the whole system is not zero E(losses)=E(1-1/x)^x=E.e^-1 and the efficiency E(efficiency)=E(1-e^-1). Similarly, we could consider the efficiency of electric field lines acting on gravitational field lines, where the efficiency is zero, but if their number (intensity ) approaches infinity , then it may not be zero.

    • @RadoslavFicko
      @RadoslavFicko 5 місяців тому

      Otherwise, a similar concept can be seen in e.g. the equations of thermal diffusion, the Yukawa potential...etc.

  • @Bethos1247-Arne
    @Bethos1247-Arne 5 місяців тому

    in fact, The Fact is a fact. It affects any student doing calculus and effective to solve such limits.

  • @BOT-mx8wq
    @BOT-mx8wq 5 місяців тому

    Professor, there is a problem that when converting the variable x to t, when x tends to infinity, the value of t should also be 1/3t.

  • @ryanswob3583
    @ryanswob3583 3 місяці тому

    If e is about 2.7 lets make feb 7th national e day (just like pi day) where we bring in eclaires to class

  • @henrygreen2096
    @henrygreen2096 5 місяців тому +2

    I do understand most of it, but it does still bother me that we 1/inf doesn’t simplify to zero. Normally I’d let it go, but I feel like there were situations where that was okay to do.
    And then to top it off “exactly 1^x” is okay to work with? When x approaches infinity? Hmm. I will have to go back to the drawing board for this one! But that you for bringing this to my attention again!
    Edit: absolutely terrible phrasing. And thank you to everyone replying! I understand now. Beautiful explanations.

    • @F1r1at
      @F1r1at 5 місяців тому +4

      Thing is 0 (as well as ∞) in that case is not a number. It's a variable like a t in the video. It just holds really smal number, which is almost an actual 0, but never exactly it.
      So while you can do 1/∞ = 0, you can't do 1+0 = 1, cause you're not adding 2 plain numbers, you're adding a number and a variable.
      1 + 0 is only approximately 1, which is fine, if there are no further operations that could change this 0 into something other, then just a really small number, but with a (1+0)^∞ you can't do that, cause ^∞ does change it.
      It's fine because the result of limits is often an approximate value which the actual result approaches, but never equal to.
      As for 1^∞ where 1 is exactly 1, yes, it is 1, because you can multiply 1 by 1 how many times you wan't, it will not change the answer.

    • @phiefer3
      @phiefer3 5 місяців тому +1

      The general replacement of 1/inf = 0 as well as 1/0 = infinity that is somewhat common in calculus, is that these are shorthands that assume that this is the only relevant behavior of the function. But in this case that's not true. We have 1/inf that is pushing the function towards 1, but there's also the exponent that is pushing the function towards infinity. Because these two effects are competing with eachother we cannot adequately describe the overall behavior of the function with this simple replacement. We technically *can* make the substitution, but as noted in the video we end up with 1^inf which is indeterminate and so doesn't help us to evaluate the limit here. The substitution isn't "wrong" it's just not useful.
      As far as the "exactly 1"^inf part, this is also a bit of a shorthand that doesn't fully explain the reasoning. 1^inf is indeterminate, in all situations. However, as is often the case in calculus, we can still evaluate indeterminate forms within the context of the function/limit we are working with. And as it happens, when the function you are working with is simply 1^x as x->infinity, this limit does evaluate to 1. The way he phrases it with the "exactly 1" wording is a little misleading (though not wrong) in that it sort of hides the actual reason. The actual reason is because it's the evaluation of a specific limit that gives us 1, whereas other functions (such as the one in this video) are different functions where 1^inf may evaluate to a different value.
      It's the same as how x/x as x->inf is an inf/inf situation that evaluates to 1 in the limit, but other inf/inf limits for other functions may end up having different values.

    • @roerd
      @roerd 5 місяців тому

      It would simplify to zero if that was the last step in calculating the limit. But since it's not in this case, but rather an intermediate step in calculating the limit, you have to take into account that the later calculations that will be performed on this intermediate step will so-to-say "cancel out" the effect.

    • @foty7
      @foty7 5 місяців тому +1

      From my perspective, the crux of the issue is that the x in the denominator and in the exponent are the *same* variable, which can interfere with intuition. Two important things to remember: (1) the limit to infinity is defined by what happens to the estimates as we plug in larger and larger values; we can never actually plug in infinity, and even if we could, we actually don’t care what it would be! We only care about the behavior of the values as we get “close” to infinity. (2) We *cannot* increase one x before or without increasing the other-they are the same x.
      #1 is a definition, so that’s not changeable, but what if we break #2 by forcibly separating them? It actually gets pretty fun: Getting rid of the 3 for clarity, let’s say we have f(x,y) = (1 + 1/x)^y … Then the limit would actually depend on *how* we send x and y to infinity-graphically, you can think of this as following some path in 2D out into quadrant 1, and we can pick the path freely so long as it goes on forever in that quadrant!
      If we follow the limit of (x,y) -> (inf,inf) along the line y=x, then we get the classic limit of e. If we follow it along y=x/3, we get e^(1/3) like in the video, and in general, for y=mx+b, I’m pretty sure we get e^m.
      In the special case of evaluating strictly x first then y, or vice versa, the graphical model gets disconnected from the limit a bit, but you can imagine it as following along the x or y axis to infinity in that direction, and then after we “reach” infinity along that axis, we then turn 90° and going in the the other axis’ direction toward infinity.
      Along x axis: This is doing x -> inf first, then y -> inf, which gets you 1^y -> 1 for the limit. (You can get this even by just plugging in 0 for m in e^m above.)
      Along the y axis: This is doing y -> inf first, which… uhhh, I think diverges to + or - infinity? (Either way, this is an edge case.)

    • @henrygreen2096
      @henrygreen2096 5 місяців тому

      Thank you so much everyone for helping me Understand this!

  • @aryanbhaiyt295
    @aryanbhaiyt295 Місяць тому

    Hey!! This type of problem having a formula bro just use it up

  • @clintonweir7609
    @clintonweir7609 5 місяців тому

    hmm.
    [1 + 1/(3x)]^x = [(3x+1)/3x]^x
    As x approaches inf, 3x+1 approaches 3x, and so (3x+1)/(3x) approaches 1, and so the whole expression approaches 1.
    Okay, prolly not. You probably need an expansion.

  • @DonTheRealMan
    @DonTheRealMan 4 місяці тому

    Is it just me or were there police sirens in the background

  • @stephanefoutel9432
    @stephanefoutel9432 Місяць тому +1

    306 / 5 000
    hello, I love what you do however I would like you to add something more to your mathematics presentations such as the possible and applicable use of your equation in a specific field like chemistry, measurements, probability or related mathematics to our daily life, have a good day

  • @SOUPARNOAdhikari-do8ib
    @SOUPARNOAdhikari-do8ib 5 місяців тому +31

    Just Log it out.

    • @dathaniel9403
      @dathaniel9403 5 місяців тому +3

      Yep. The limit of (1+1/x)^x as x->infinity is e≈2.718, so the limit of (1+1/3x)^x is e^(1/3)

    • @47_PrincE
      @47_PrincE Місяць тому

      Yes I was thinking about that too

  • @alphalunamare
    @alphalunamare 5 місяців тому

    6:45 Is it actually 'legal' to shift the cube root outside of the limit? That was raher slight of hand. The very thing that you are trying to expose as being wrong.

    • @Rafaeu777
      @Rafaeu777 5 місяців тому

      Perfectly legal, cube root of x is continuous on every point of the line, in particular on R+, so yes. It could even be a square root, no problem in that specific limit.

    • @alphalunamare
      @alphalunamare 5 місяців тому

      @@Rafaeu777 True, but is this limit finite? If not then all bets are off.

    • @Rafaeu777
      @Rafaeu777 5 місяців тому

      @@alphalunamare Not really, you can use this same argument along with the extension of Infinite limits and limits at infinity

  • @siavashghazisaidi8338
    @siavashghazisaidi8338 3 місяці тому +1

    In limits,there are two types of numbers:absolute and relative. Relative 1 to the power of infinity is indeterminate. Absolute 1 to the power of anything,even infinity, is 1. We must always keep this in mind.

  • @user-rb5wn3xk9i
    @user-rb5wn3xk9i 2 місяці тому +1

    just go back to the ε-δ definition then everything are clear

  • @gabro2451
    @gabro2451 4 місяці тому +2

    have I passed calculus several years ago? yes. have I already knew the answer? yes. have I watched the video till the end because of the brilliant and easy explanation? also yes.

  • @GuniDubey-rz2gu
    @GuniDubey-rz2gu 5 місяців тому +4

    I thought it is 1

  • @uartim
    @uartim 2 місяці тому

    Compound rate.

  • @user-nd7th3hy4l
    @user-nd7th3hy4l 5 місяців тому

    1/3

  • @bertblankenstein3738
    @bertblankenstein3738 2 місяці тому

    OK my idea was slightly different. I substituted 3x with n. Crazy i know.

  • @RobertGabor
    @RobertGabor 3 місяці тому

    Answer is on picture on right "e" or similar ;) (1+1/3x)^3x = e but how to aproxomate e^(⅓)…?

  • @voidmxl8473
    @voidmxl8473 5 місяців тому

    FACTS (literally) :D

  • @Kambyday
    @Kambyday 5 місяців тому

    Hey, can you find out why this proof of 1/∞=0?
    (1/∞)+1=x
    1+∞/∞=x
    ∞/∞=x
    x=1
    Now back to our original equation
    (1/∞)+1=1
    Subtracting 1 from both sides
    1/∞=0

    • @thenetsurferboy
      @thenetsurferboy 5 місяців тому

      Think if you can.
      1/5 = 0.2
      1/100 = .01
      1/1000 = 0.001
      1/10000 = 0.0001
      See, getting closer to 0
      1/infinity = 0. 00 to an infinite number of 0/s

    • @Kambyday
      @Kambyday 5 місяців тому

      @@thenetsurferboy yes you would be correct but mathematically 1/∞ is infinitely small but not zero, we just approximate as zero since it's so close, actually the closest thing to zero ever

  • @lindenjenesse5078
    @lindenjenesse5078 5 місяців тому

    I love you 3000!

  • @person1082
    @person1082 5 місяців тому

    ln(1^infty)=infty ln(1)=0 infty

  • @RexxSchneider
    @RexxSchneider 5 місяців тому

    No. lim_t→∞(1 + 1/t)^t is not e BY DEFINITION, but by result. Don't they teach the binomial expansion any more?
    (1+ a)^n = 1 + na + n(n-1)a^2/2! + n(n-1)(n-2)a^3/3! + ... Set n = 1/a = t and you get:
    (1 + 1/t)^t = 1 + 1 + t(t-1)/t^2/2! + t(t-1)(t-2)/t^3/3! + .. As t→∞, we can see that t ≈ (t-1) ≈ (t-2), etc. Therefore t(t-1)/t^2 ≈ 1, and t(t-1)(t-2)/t^3 ≈ 1, etc. So we get:
    lim_t→∞(1 + 1/t)^t = 1 + 1 + 1/2! + 1/3! +1/4! which is e _by definition._

    • @notmymain2256
      @notmymain2256 5 місяців тому +1

      There are two main definitions of e which are equivalent (and that's a theorem you can prove), one with the power series and one as the limit of the *sequence* a_n = (1+1/n)^n as n->infty. It's important to notice it's the limit of a sequence and not of the function (1+1/x)^x as exponentiation with real exponents is only defined through the exp function (basically after defining e and proving some properties)

    • @notmymain2256
      @notmymain2256 5 місяців тому

      Also, beware of "substituting" t(t-1)...(t-k+1)/t^k with 1 as, while it would be legal in a limit of a sequence by the theorem of substitution (not sure about the name, I learnt about it in another language) here you're considering an infinite sum, and as you probably know the infinite sum of infinitesimal errors can be non infinitesimal (well, in this case it still works, but you need to put more work to prove it)

    • @RexxSchneider
      @RexxSchneider 5 місяців тому +1

      @@notmymain2256 The most distinctive definition in my opinion is the base of logarithms whose anti-derivative is 1/x. But of course, there are lots of "definitions" that can be derived from each other. Nevertheless, it is rarely helpful to start from the limit as you suggest, since it is trivially found equivalent to the power series by use of the binomial theorem.
      The limit is, of course, the same whether it is approached through the integers or the reals. Your point about exponentiation with reals is another reason why using limits is a less satisfactory starting point for a definition.

  • @polygondeath2361
    @polygondeath2361 3 місяці тому +1

    Do you need to know this for ap calc bc?

    • @lugia8888
      @lugia8888 2 місяці тому +1

      What a lazy student

  • @user-vs0s1su4ka
    @user-vs0s1su4ka 5 місяців тому +2

    When I see such questions, the only thought I have: why did you decide that it can be replaced?

    • @geirmyrvagnes8718
      @geirmyrvagnes8718 5 місяців тому

      Practice and memory. Then you recognize patterns. "This looks like the formula for e, can I use that?". Some math youtube channels skip this part, and just say "let us write this a different way for no reason". That is not how it works. This channel is much better than most in explaining WHY you do those first interesting and sometimes creative steps. The rest is following the rules and seeing where you end up.

  • @JJ_TheGreat
    @JJ_TheGreat 5 місяців тому

    4:42 Prove that that is equal to e ! 😀

  • @SonEdison-xk1mp
    @SonEdison-xk1mp 2 місяці тому +1

    I'd like to say this one is way too easy

    • @operamiser
      @operamiser Місяць тому +1

      Stop being toxic and cause frustration to others ... this channel is about the basics... the starter ..... and he is doing very well by choosing these examples

  • @sankaranbs4024
    @sankaranbs4024 2 місяці тому

    Sir, you said that x = (1/3)t, but when substituting t in the limit,i.e., in x -----> infinity you should have been wrote it like (1/3)t -----> intinity. Why sir you wrote it as t -----> infinity

    • @lugia8888
      @lugia8888 2 місяці тому

      Dont worry about it

  • @reyallen3108
    @reyallen3108 3 місяці тому +2

    why a=1/3? xd

    •  2 місяці тому

      1/3x = (1/3)/x

  • @junkcuber5611
    @junkcuber5611 5 місяців тому

    I should have just made a YTChannel of me doing all the Calculus and AP problems over the last 20 years… maybe I could retire, lol.

    • @stephenbeck7222
      @stephenbeck7222 5 місяців тому +2

      Lots of those out there but only a few have enough viewership to make any real money. And the ones that do, spend a lot of time on their speaking craft and video design/editing. So it’s not just free money.

    • @junkcuber5611
      @junkcuber5611 5 місяців тому

      @@stephenbeck7222 Maybe, but a lot of these videos seem to be pretty low-budget all done in 1 or 2 shots. And getting a 20-year head start on it would have helped as well.

  • @carultch
    @carultch 5 місяців тому +1

    Teeth power!

  • @tusharswain3102
    @tusharswain3102 3 місяці тому

    Is 1/(infinity) indeterminate ?

    • @lugia8888
      @lugia8888 2 місяці тому

      No, it is 0. Indeterminate are like 0/0

  • @lucascaldasdecarvalhoferre5757
    @lucascaldasdecarvalhoferre5757 5 місяців тому

    ♾️-♾️ = indetermination ?
    Why ???
    The other ones in the Side of the cat I understand

    • @julioaurelio
      @julioaurelio 5 місяців тому

      Consider the limit as x approaches infinity of the functions F(x) = x^3 - x, the function G(x) = x - 2x, and the function D(x) = (x)^2 - (x+1)^2.
      In all these cases, by substituting x with infinity, you'd get "infinity - infinity", but once evaluated, it is revealed that the first limit is +Infinity, the second is -Infinity, and the third is 0.

    • @lucascaldasdecarvalhoferre5757
      @lucascaldasdecarvalhoferre5757 5 місяців тому

      @@julioaurelio hum 🧐
      I got it

  • @markmajkowski9545
    @markmajkowski9545 5 місяців тому

    What happened to “my best friend”?